Will you cube each term?

Algebra Level 4

( x 2 + x 2 ) 3 + ( 2 x 2 x 1 ) 3 = 27 ( x 2 1 ) 3 \large (x^{2} + x-2)^3 + (2x^2 - x -1)^{3} = 27(x^2-1)^{3}

Find the sum of all distinct real roots of x x satisfying the above equation.


Source: Regional Mathematics Olympiad, India.


The answer is -2.5.

This section requires Javascript.
You are seeing this because something didn't load right. We suggest you, (a) try refreshing the page, (b) enabling javascript if it is disabled on your browser and, finally, (c) loading the non-javascript version of this page . We're sorry about the hassle.

8 solutions

Department 8
Sep 24, 2015

Let a = x 2 + x 2 = ( x + 2 ) ( x 1 ) a=x^{2}+x-2=(x+2)(x-1) and b = 2 x 2 x 1 = ( 2 x + 1 ) ( x 1 ) b=2x^{2}-x-1=(2x+1)(x-1)

a 3 + b 3 = ( a + b ) 3 a b ( a + b ) = 0 ( x + 2 ) ( x 1 ) ( 2 x + 1 ) ( x 1 ) ( x 2 1 ) = 0 x = 1 , 1 , 2 , 1 2 a^{3}+b^{3}=(a+b)^{3} \\ab(a+b)=0 \\(x+2)(x-1)(2x+1)(x-1)(x^{2}-1)=0\\ x=1,-1,-2,\frac{-1}{2}

According to the question answer is 1 1 2 0.5 = 2.5 \boxed{1-1-2-0.5=-2.5}

i am a total idiot.not worthyto be a fan of iron man

Kaustubh Miglani - 5 years, 8 months ago

nice solution!!

Aareyan Manzoor - 5 years, 8 months ago

Did it the exact same way!.

A Former Brilliant Member - 5 years, 8 months ago

Good solution....... I think the problem is a bit overrated

Arnav Das - 5 years, 8 months ago

Log in to reply

Yes, I marked it level 2 at first but I guess a moderator changed it to level 4.

A Former Brilliant Member - 5 years, 8 months ago

Log in to reply

Yes, it's quite overrated. By the way, is it really a RMO's question??

Raushan Sharma - 5 years, 8 months ago

Log in to reply

@Raushan Sharma Yes it is a RMO question.

A Former Brilliant Member - 5 years, 8 months ago

Log in to reply

@A Former Brilliant Member Which year's?

Raushan Sharma - 5 years, 8 months ago

Log in to reply

@Raushan Sharma It is from RMO year 2002.

A Former Brilliant Member - 5 years, 8 months ago

I cant even believe that it is from RMO...

Mohit Gupta - 5 years, 6 months ago

well done next batman

Kaustubh Miglani - 5 years, 8 months ago

Log in to reply

Thanks next iron man.

Department 8 - 5 years, 8 months ago

Same way! . Nice solution bro

Shreyash Rai - 5 years, 5 months ago

Log in to reply

Hi! What's up, What is your plan for new year?

Department 8 - 5 years, 5 months ago

Hey Check out my name in is in top solution

Department 8 - 5 years, 5 months ago

Log in to reply

Nice. Though I will never know the solution cause there no way I can solve that

Shreyash Rai - 5 years, 5 months ago

Log in to reply

@Shreyash Rai It's okay :-)

Department 8 - 5 years, 5 months ago

Did the same way

Manit Kapoor - 5 years, 8 months ago
Chew-Seong Cheong
Sep 23, 2015

( x 2 + x 2 ) 3 + ( 2 x 2 x 1 ) 3 = 27 ( x 2 1 ) 3 ( x 1 ) 3 ( x + 2 ) 3 + ( x 1 ) 3 ( 2 x + 1 ) 3 = 27 ( x 1 ) 3 ( x + 1 ) 3 \begin{aligned} (x^2+x-2)^3+(2x^2-x-1)^3 & = 27(x^2-1)^3 \\ (x-1)^3(x+2)^3 + (x-1)^3(2x+1)^3 & = 27(x-1)^3 (x+1)^3 \end{aligned}

We note that x = 1 x = 1 is a root and that:

( x + 2 ) 3 + ( 2 x + 1 ) 3 = 27 ( x + 1 ) 3 = ( 3 x + 3 ) 3 = ( x + 2 + 2 x + 1 ) 3 a 3 + b 3 = ( a + b ) 3 a b ( a + b ) = 0 ( x + 2 ) ( 2 x + 1 ) ( 3 x + 3 ) = 0 ( x + 2 ) ( 2 x + 1 ) ( x + 1 ) = 0 \begin{aligned} (\color{#3D99F6}{x+2})^3 + (\color{#D61F06}{2x+1})^3 & = 27(x+1)^3 = (3x+3)^3 = (\color{#3D99F6}{x+2} +\color{#D61F06}{2x+1})^3 \\ \Rightarrow \color{#3D99F6}{a}^3 + \color{#D61F06}{b}^3 & = (\color{#3D99F6}{a} + \color{#D61F06}{b})^3 \\ \Rightarrow ab(a+b) & = 0 \\ (x+2)(2x+1)(3x+3) & = 0 \\ \Rightarrow (x+2)(2x+1)(x+1) & = 0 \end{aligned}

This implies that all the four real roots are 1 1 , 1 -1 , 2 -2 and 1 2 -\frac{1}{2} and their sum is 2.5 \boxed{-2.5} .

Not that when cube roots of the terms are added they equal 0 which means that the expression will equal three times their product which is also 0 so we will get the answer more easily there.

Kushagra Sahni - 5 years, 8 months ago

Log in to reply

I don't understand what you are saying. Please clarify.

Alan Yan - 5 years, 8 months ago

I think you are saying ( x + 2 ) 3 + ( 2 x + 1 ) 3 27 ( x + 1 ) 3 = 0 (x+2)^3+(2x+1)^3-27(x+1)^3=0 . So, if x + 2 = 0 x+2 = 0 , 2 x + 1 = 0 2x+1=0 and x + 1 = 0 x+1=0 then the whole expression is equal to 0 0 . But when x + 2 = 0 x+2=0 , (2x+1= -3) and x + 1 = 1 x+1=-1 .

Chew-Seong Cheong - 5 years, 8 months ago

Log in to reply

If we define a = x + 2 , b = 2 x + 1 a = x+2, b = 2x+1 , then a + b = 3 ( x + 1 ) a+b = 3(x+1) . Therefore, the given condition becomes a 3 + b 3 = ( a + b ) 3 a^3 + b^3 = (a+b)^3 or equivalently a b ( a + b ) = 0 ab(a+b) = 0 . This is a simple way to jump from step #3 to step #10 in your derivation.

Kaushik Chakraborty - 5 years, 8 months ago

Log in to reply

@Kaushik Chakraborty Thanks. I have failed see this point and also to read others' solutions.

Chew-Seong Cheong - 5 years, 8 months ago

Log in to reply

@Chew-Seong Cheong I was saying what Lakshay Sinha did

Kushagra Sahni - 5 years, 8 months ago

Same way I did

Atul Shivam - 5 years, 8 months ago

Note first that if we were to expand all the terms we would get a degree 6 6 polynomial, so we can expect 6 6 solutions, although we cannot tell at this point if all of them will be real.

Now the equation can be rewritten as ( ( x 1 ) ( x + 2 ) ) 3 + ( ( 2 x + 1 ) ( x 1 ) ) 3 = 3 3 ( ( x 1 ) ( x + 1 ) ) 3 . ((x - 1)(x + 2))^{3} + ((2x + 1)(x - 1))^{3} = 3^{3}((x - 1)(x + 1))^{3}.

Thus x = 1 x = 1 is a root of multiplicity 3 , 3, and so we are looking for 3 3 more roots, (not necessarily all real). Dividing through by ( x 1 ) 3 (x - 1)^{3} gives us the equation

( x + 2 ) 3 + ( 2 x + 1 ) 3 = ( 3 x + 3 ) 3 . (x + 2)^{3} + (2x + 1)^{3} = (3x + 3)^{3}.

Now we could play a hunch and suppose that ( x + 2 ) , ( 2 x + 1 ) (x + 2), (2x + 1) and ( 3 x + 3 ) (3x + 3) are all integers, in which case, by Fermat's Last Theorem, not all of them can be positive integers, (or all negative integers, for that matter). So if we were to successively set each of these terms equal to 0 , 0, we quickly notice that x = 2 , x = 1 2 x = -2, x = -\frac{1}{2} and x = 1 x = -1 all satisfy this last equation. This indicates that we have found all the real roots, the distinct roots summing to 1 + ( 1 ) + ( 2 ) + ( 1 2 ) = 5 2 = 2.5 . 1 + (-1) + (-2) + (-\frac{1}{2}) = -\frac{5}{2} = \boxed{-2.5}.

Less intuitively, we can expand the last equation to end up with

x 3 + 6 x 2 + 12 x + 8 + 8 x 3 + 12 x 2 + 6 x + 1 = 27 x 3 + 81 x 2 + 81 x + 27 x^{3} + 6x^{2} + 12x + 8 + 8x^{3} + 12x^{2} + 6x + 1 = 27x^{3} + 81x^{2} + 81x + 27

18 x 3 + 63 x 2 + 63 x + 18 = 0 x 3 + 7 2 x 2 + 7 2 x + 1 = 0. \Longrightarrow 18x^{3} + 63x^{2} + 63x + 18 = 0 \Longrightarrow x^{3} + \dfrac{7}{2}x^{2} + \dfrac{7}{2}x + 1 = 0.

Now by Vieta's we know that the sum of the remaining roots is 7 2 , -\dfrac{7}{2}, but we don't know yet if all the remaining roots are real. By Descartes' Rule of Signs we know that none of the remaining roots are positive and that either 3 3 or 1 1 are negative. However, by the symmetry of this last equation we see that x = 1 x = -1 is a solution, allowing us to further factor to

( x + 1 ) ( x 2 + 5 2 x + 1 ) = ( x + 1 ) ( x + 1 2 ) ( x + 2 ) = 0 , (x + 1)(x^{2} + \frac{5}{2}x + 1) = (x + 1)(x + \frac{1}{2})(x + 2) = 0,

which confirms that the remaining roots are all negative reals which sum to 7 2 . -\dfrac{7}{2}. Adding the root x = 1 x = 1 to this gives the same answer as before, namely 1 7 2 = 5 2 = 2.5 . 1 - \dfrac{7}{2} = -\dfrac{5}{2} = \boxed{-2.5}.

First time I have ever seen Fermat's Last Theorem quoted in a proof.

Terry Smith - 5 years, 8 months ago

Log in to reply

Haha. Yeah, just for fun, I was trying to mention as many names as possible in this solution. :)

Brian Charlesworth - 5 years, 8 months ago

Great job!

A Former Brilliant Member - 5 years, 8 months ago
Rwit Panda
Sep 26, 2015

It's easy to observe that if each of the expression which is cubed is taken and added, then the result is 0. We know that when a+b+c=0, then a^3 + b^3 + c^3= 3abc.

Also, here a^3 + b^3 + c^3 = 0. So, 3abc=0. This enables us to factor the given terns and put the whole thing = 0. So we have distinct solutions of 1,-1,-2,-0.5 .So the sum is -2.5

( x 2 + x 2 ) 3 + ( 2 x 2 x 1 ) 3 + ( 3 3 x 2 ) 3 = 0 (x^{2}+x-2)^{3}+(2x^{2}-x-1)^{3}+(3-3x^{2})^{3}=0 As ( x 2 + x 2 ) + ( 2 x 2 x 1 ) + ( 3 3 x 2 ) = 0 (x^{2}+x-2)+(2x^{2}-x-1)+(3-3x^{2})=0 we have that the inital ecuation becomes 9 ( x 2 + x 2 ) ( 2 x 2 x 1 ) ( 1 x 2 ) = 0 9(x^{2}+x-2)(2x^{2}-x-1)(1-x^{2})=0 where the distinct solutions are 1 ; 1 ; 2 ; 1 2 1;-1;-2;-\frac{1}{2}

Rupesh Kapu
Oct 5, 2015

fermat's last theorem states that "there is no non trivial solution to the equation a^n+b^n=c^n for n>=3",hence for the above equation we have a=0,b=0,c=0,therefore on solving we get x=1,-1,-2,-0.5 =>-2.5

Atul Shivam
Oct 5, 2015

Upon factorising the above term we get roots which are 1 , 1 2 , 2 , 1 a n d 1 1, -\frac {1}{2},-2,-1and 1 out of which we have to find the sum of 1 , 1 2 , 2 a n d 1 1 , -\frac {1}{2},-2and-1 which is -2•5 as we have to take distinct roots we have neglected 1,1

Madhurima Gupta
Sep 23, 2015

Let a= x 2 + x 2 x^{2}+x-2 b= 2 x 2 x 1 2x^{2}-x-1 c= 3 ( x 2 1 ) -3 (x^{2}-1) Then a+b+c= x 2 + x 2 + ( 2 x 2 x 1 ) + ( 3 x 2 3 ) x^{2} +x-2 +(2x^{2}-x-1)+(-3x^{2}-3) = 0

Hence a 3 + b 3 + c 3 3 a b c a^{3}+b^{3}+c^{3} -3abc = (a+b+c) a 2 + b 2 + c 2 a b b c c a a^{2} +b^{2}+ c^{2}-ab-bc-ca =0

Hence a 3 + b 3 + c 3 a^{3} +b^{3}+c^{3} = 3abc = 0

   3(x^{2}+x-2)(2x^{2}-x-1)(-3x^{2} -3x) =0

Hence x= 1-1-2-0.5 = -2.5

0 pending reports

×

Problem Loading...

Note Loading...

Set Loading...